Statistics Data Sufficiency

This topic has expert replies
Master | Next Rank: 500 Posts
Posts: 137
Joined: Fri Nov 13, 2015 11:01 am
Thanked: 1 times
Followed by:2 members

Statistics Data Sufficiency

by Amrabdelnaby » Mon Nov 16, 2015 4:40 am

Timer

00:00

Your Answer

A

B

C

D

E

Global Stats

Could you please help me with this one?

Beginning in January of last year, Carl made deposits of $120 into his account on the 15th of each month for several consecutive months and then made withdrawals of $50 from the account on the 15th of each of the remaining months of last year. There were no other transactions in the account last year. If the closing balance of Carl's account for May of last year was 2,600, what was the range of the monthly closing balances of Carl's account last year?

1- Last year the closing balance of Carl's account for April was less than $2,625
2- Last year the closing balance of Carl's account for June was less than $2,675

User avatar
GMAT Instructor
Posts: 15539
Joined: Tue May 25, 2010 12:04 pm
Location: New York, NY
Thanked: 13060 times
Followed by:1906 members
GMAT Score:790

by GMATGuruNY » Mon Nov 16, 2015 6:15 am
Beginning in January of last year, Carl made deposits
of $120 into his account on the 15th of each month
for several consecutive months and then made
withdrawals of $50 from the account on the 15th of
each of the remaining months of last year. There were
no other transactions in the account last year. If the
closing balance of Carl's account for May of last year
was $2,600, what was the range of the monthly
closing balances of Carl's account last year?

(1) Last year the closing balance of Carl's account for April was less than $2,625.
(2) Last year the closing balance of Carl's account for June was less than $2,675.
May = 2600.

Statement 1: Last year the closing balance of Carl's account
for April was less than $2,625.

If 120 was deposited in May, then April = 2600-120 = 2480.
If 50 was withdrawn in May, then April = 2600+50 = 2650.
Since April < 2625, we know that a deposit of 120 was made in May, implying that 120 was deposited every month January through at least May.
Thus, we can determine the balance of every month January - April:
April = 2480, March = 2480-120 = 2360, etc.
No information about June - December.
Insufficient.

Statement 2: Last year the closing balance of Carl's account
for June was less than $2,675.

If 120 was deposited in June, then June = 2600+120 = 2720.
If 50 was withdrawn in June, then June = 2600-50 = 2550.
Since June < 2675, we know that a withdrawal of 50 was made in June, implying that 50 was withdrawn every month June - December.
Thus, we can determine the balance of every month June - December:
June = 2550, July = 2550-50 = 2500, etc.
No information about January - April.
Insufficient.

Statements 1 and 2 combined:
Statement 1 tells us the closing balances January - April.
The question stem tells us the closing balance in May.
Statement 2 tells us the closing balances June - December.
Combining the information above, we can determine the closing balance of every month and determine the range for the whole year.
Sufficient.

The correct answer is C.
Private tutor exclusively for the GMAT and GRE, with over 20 years of experience.
Followed here and elsewhere by over 1900 test-takers.
I have worked with students based in the US, Australia, Taiwan, China, Tajikistan, Kuwait, Saudi Arabia -- a long list of countries.
My students have been admitted to HBS, CBS, Tuck, Yale, Stern, Fuqua -- a long list of top programs.

As a tutor, I don't simply teach you how I would approach problems.
I unlock the best way for YOU to solve problems.

For more information, please email me (Mitch Hunt) at [email protected].
Student Review #1
Student Review #2
Student Review #3

GMAT/MBA Expert

User avatar
GMAT Instructor
Posts: 16207
Joined: Mon Dec 08, 2008 6:26 pm
Location: Vancouver, BC
Thanked: 5254 times
Followed by:1268 members
GMAT Score:770

by Brent@GMATPrepNow » Mon Nov 16, 2015 7:36 am
Beginning in January of last year, Carl made deposits of $120 into his account on the 15th of each month for several consecutive months and then made withdrawals of $50 from the account on the 15th of each of the remaining months of last year. There were no other transactions in the account last year. If the closing balance of Carl's account for May of last year was $2,600, what was the range of the monthly closing balances of Carl's account last year?

(1) Last year the closing balance of Carl's account for April was less than $2,625
(2) Last year the closing balance of Carl's account for June was less than $2,675
Target question: What was the range of the monthly closing balances of Carl's account last year?

Given: The closing balance of Carl's account for May of last year was $2,600

IMPORTANT: To answer the target question we need only determine which month Carl STARTED withdrawing money.
For example, if he started withdrawing money on March 15, we could use the fact that he had $2600 at the end of May to determine how much he had in the bank every month of the year, and thus determine the range of closing balances.
Notice that, since this is a Data Sufficiency, we need not calculate the actual range. We need only determine which month the deposits stopped and the withdrawals started.

So, we can rephrase our target question as . . .

REPHRASED target question: In which month did Carl start withdrawing $50?

Statement 1: Last year the closing balance of Carl's account for April was less than $2,625
Let's examine two cases:
case a: In May, Carl DEPOSITED $120. So, balance at end of April = $2600 - $120 = $2480. This is possible, since we're told that the balance is less than $2625
case b: In May, Carl WITHDREW $50. So, balance at end of April = $2600 + $50 = $2650. This is NOT possible, since we're told that the balance is less than $2625
So, Carl definitely deposited $120 in May (and deposited $120 in April, March, Feb, and Jan).
However, we don't know the first month that Carl started withdrawing $50
Since we cannot answer the REPHRASED target question with certainty, statement 1 is NOT SUFFICIENT

Statement 2: Last year the closing balance of Carl's account for June was less than $2,675.
Let's examine two cases:
case a: In June, Carl DEPOSITED $120. So, balance at end of June = $2600 + $120 = $2720. This is NOT possible, since we're told that the balance is less than $2675
case b: In June, Carl WITHDREW $50. So, balance at end of June = $2600 - $50 = $2550. This is possible, since we're told that the balance is less than $2675
So, Carl definitely withdrew $50 in June, which means he also withdrew $50 in July, August, Sept, etc. However, we don't know the FIRST month that Carl started withdrawing $50
Since we cannot answer the REPHRASED target question with certainty, statement 2 is NOT SUFFICIENT

Statements 1 and 2 combined
Statement 1 tells us that Carl deposited $120 in May.
Statement 2 tells us that Carl withdrew $50 in June.
So, June was the first month that Carl started withdrawing $50
Since we can answer the REPHRASED target question with certainty, the combined statements are SUFFICIENT

Answer = C

Cheers,
Brent
Brent Hanneson - Creator of GMATPrepNow.com
Image

User avatar
MBA Admissions Consultant
Posts: 2279
Joined: Fri Nov 11, 2011 7:51 am
Location: New York
Thanked: 660 times
Followed by:266 members
GMAT Score:770

by Jim@StratusPrep » Mon Nov 16, 2015 10:45 am
You will find this pdf helpful:
Attachments
OG 13 DS 143.pdf
(1.66 MiB) Downloaded 133 times
GMAT Answers provides a world class adaptive learning platform.
-- Push button course navigation to simplify planning
-- Daily assignments to fit your exam timeline
-- Organized review that is tailored based on your abiility
-- 1,000s of unique GMAT questions
-- 100s of handwritten 'digital flip books' for OG questions
-- 100% Free Trial and less than $20 per month after.
-- Free GMAT Quantitative Review

Image